Peskin & Schroeder: Freie Teilchenausbreitung

In Peskin & Schroeder Ch. 2, p. 14, indem sie beweisen, dass die NRQM-Ausbreitungsamplitude für ein freies Teilchen überall ungleich Null ist, gehen sie davon aus

U ( T )   =   1 ( 2 π ) 3 D 3 P e ich ( P 2 / 2 M ) T e ich P ( X X 0 )
zum Endergebnis:
U ( T )   =   ( M 2 π ich T ) 3 / 2 e ich M ( X X 0 ) 2 / 2 T .

Ich verstehe nicht alle Schritte dazwischen. Bei der Auswertung des ersten Integrals habe ich es zuerst in Polarkoordinaten gesetzt, zusammen mit z X X 0 , aber dann lande ich schließlich bei einer Gaußschen Integration, die so aussieht, als sollte sie Null sein. Wie komme ich von der ersten Gleichung zur zweiten?

BEARBEITEN:

Der nächste Schritt, den ich nach dem obigen mache, ist, das Integral neu zu schreiben:

1 ( 2 π ) 2 0 1 1 D P D cos θ P 2 e ich ( P 2 / 2 M ) T e ich P Δ X cos θ

Wo Δ X | X X 0 | und ich habe die Integration über ϕ . Von hier bekomme ich

1 ( 2 π ) 2 ich Δ X 0 D P P e ich ( P 2 / 2 M ) T ( e ich P Δ X e ich P Δ X ) .

Es scheint, als ob diese Integration geben sollte 0 , es sei denn, ich mache irgendwo einen Fehler. Wo ist mein Fehler?

Danke – das habe ich gesehen, aber es sieht so aus, als würden sie leider erst kurz vor meiner Verwirrung auf den Punkt kommen …

Antworten (3)

Das scheinst du zu denken

1 ( 2 π ) 2 ich Δ X 0 D P P e ich ( P 2 / 2 M ) T ( e ich P Δ X e ich P Δ X ) = 0

wahrscheinlich, weil die Exponentialfunktionen so aussehen, als würden sie sich aufheben, aber das ist nicht so. Beachten Sie, dass

e ich P Δ X e ich P Δ X = 2 ich Sünde ( P Δ X )

was bedeutet, dass Sie berechnen möchten

2 ( 2 π ) 2 Δ X 0 D P P e ich ( P 2 / 2 M ) T Sünde ( P Δ X )

und es ist ganz offensichtlich, dass dies nicht verschwinden wird. Tatsächlich können wir am zweiten Term des ursprünglichen Integrals ein wenig arbeiten

1 ( 2 π ) 2 ich Δ X 0 D P P e ich ( P 2 / 2 M ) T e ich P Δ X

Ersatz P ' = P , dann ist dies gleich

1 ( 2 π ) 2 ich Δ X 0 D P ' P ' e ich ( P ' 2 / 2 M ) T e ich P ' Δ X

Ihr ursprüngliches Integral ist also gerecht

1 ( 2 π ) 2 ich Δ X D P P e ich ( P 2 / 2 M ) T e ich P Δ X = 1 ( 2 π ) 2 ich Δ X ich D D Δ X D P e ich ( P 2 / 2 M ) T e ich P Δ X

dies ist nur die Ableitung eines normalen Gaußschen Integrals. Verwendung der allgemeinen Formel für Gaußsche Integrale

e A X 2 + B X D X = π A   e B 2 4 A

die RGJ bereits in seiner Antwort bereitgestellt hat, erhalten wir sofort

( M 2 π ich T ) 3 / 2 exp ( ich Δ X 2 M 2 T )

das ist Ihr gewünschtes Ergebnis.

Großartig, vielen Dank. Mein Problem bestand darin, den Koeffizienten von p aus dem Integral herauszubekommen, und ich hatte nicht daran gedacht, es so zu machen.
@gh3 Dies ist eigentlich ein sehr nützlicher Trick, der Differenzierung unter dem Integralzeichen genannt wird und für alle Arten von Gaußschen Integralen sehr nützlich sein wird. Ich würde empfehlen, dass Sie sich das irgendwo genauer ansehen.

Beachten Sie das Integral einer beliebigen Gaußschen Funktion,

e A X 2 + B X D X = π A   e B 2 4 A

Kleiner Kommentar: qualifiziert von A > 0 .
Ja, verstanden. Mein Problem ist, dass ich bei der Ableitung am Ende einen Faktor von habe P vor dem Gaußschen, immer noch integrierend von negativ unendlich bis positiv unendlich, was bedeutet, dass das Integral zu bewertet wird 0 .
Meine Antwort basiert auf Ihrer kurzen.
@Frobenius: Danke für die Ausarbeitung meiner Antwort. Ich dachte das würde reichen!

Hinweis :

Stellen Sie die Hypothese auf , dass das Integral einer beliebigen Gaußschen Funktion (siehe Antwort von RGJ)

(01) e A X 2 + B X D X = π A   e ( B 2 / 4 A )
gilt für A , B reine imaginäre Zahlen und für unseren Fall
(02.1) A = ich ( T 2 M ) (02.2) B k = ich ( X X 0 ) k , k = 1 , 2 , 3
Dann
(03.1) + e A P 1 2 + B 1 P 1 D P 1 = π A e ( B 1 2 / 4 A ) = 2 π M ich T e ich M | ( X X 0 ) 1 | 2 / 2 T (03.2) + e A P 2 2 + B 2 P 2 D P 2 = π A e ( B 2 2 / 4 A ) = 2 π M ich T e ich M | ( X X 0 ) 2 | 2 / 2 T (03.3) + e A P 3 2 + B 3 P 3 D P 3 = π A e ( B 3 2 / 4 A ) = 2 π M ich T e ich M | ( X X 0 ) 3 | 2 / 2 T
Wenn wir die obigen 3 Gleichungen nebeneinander multiplizieren, haben wir
(04) + + + e A P 2 + B P D P 1 D P 2 D P 3 = ( π A ) 3 / 2 e ( B 2 / 4 A ) = ( 2 π M ich T ) 3 / 2 e ich M X X 0 2 / 2 T
oder
(05) R 3 e ich ( P 2 / 2 M ) T + ich P ( X X 0 ) D 3 P = ( 2 π M ich T ) 3 / 2 e ich M X X 0 2 / 2 T
Und
(06) 1 ( 2 π ) 3 R 3 e ich ( P 2 / 2 M ) T + ich P ( X X 0 ) D 3 P = ( M 2 π ich T ) 3 / 2 e ich M X X 0 2 / 2 T
Versuchen Sie also, die Hypothese zu beweisen .